Questions sur une inégalité

Bonjour,

Si $x_1,\dots,x_n$ sont des réels, alors $\sum_{i,j} |x_i - x_j| \leq \sum_{i,j} |x_i + x_j|$.

Savez-vous :
1. Si cette inégalité porte un nom ?
2. Si cette inégalité reste vraie pour des nombres complexes ?

Remarque : il est facile de voir que $\sum_{i,j} \|x_i - x_j\| \leq 2\sum_{i,j} \|x_i + x_j\|$ est vraie en toute dimension.
«1

Réponses

  • Ça marche avec x=−1 et y=1 ?
    Algebraic symbols are used when you do not know what you are talking about.
            -- Schnoebelen, Philippe
  • Oui, parce qu'à droite, $i$ peut être égal à $j$
  • Merci de vous y intéresser. A priori l'inégalité devrait être juste, d'ailleurs vous pouvez vous amuser à la démontrer : ça devrait faire un bon exo d'oral ou d'olympiades par exemple. Le seul souci, c'est que la preuve que j'ai ne marche vraiment que dans le cas de $\Bbb R$ (d'où ma question).
  • Eh bien, si elle ne porte pas de nom, nous pourrions l'appeler "l'inégalité de Siméon, le Poisson", sauf autre référence, si tu le veux bien.

    D'où la sors-tu ?
    Pour la démonstration, je sèche encore.

    Juste cette précision : l'indiciation est elle correcte ici ?
    $$\sum_{(i,j)\in \{ 1,\dots n\} ^2 } |x_i - x_j| \leq \sum_{(i,j)\in \{ 1,\dots n\} ^2 } |x_i + x_j|$$
    C'est à dire qu'il y a $n^2$ termes de chaque côté dont au moins $n$ sont nuls à gauche.

    Amicalement. jacquot
  • bjr ,
    ceci m' intrigue , j 'ai pris 10 valeurs xi et 10 valeurs xj consécutives et l'inégalité n'est pas vraie.
    J'ai dû me planter ...
    restart;
    > S1:=0;S2:=0;
    > for x from -2 to -11 by -1 do
    > for y from 11 to 20 do
    > S1:=abs(x-y)+S1; S2:=abs(x+y)+S2 ;
    > od;od;
    > S1;S2;

    S1=2200 , S2=900
  • $|x_i+x_j|$ revient-il une deuxième fois sous la forme $|x_j+x_i|$ dans ta somme ?
  • oubliez ce qui précède; pour l'exemple , il n'y a que 10 nombres xi et xj en tout et pas 20 !

    restart;
    > S1:=0;S2:=0;
    > for x from 1 to 10 do
    > for y from 1 to 10 do
    > S1:=abs(x-y)+S1; S2:=abs(x+y)+S2 ;
    > od;od;
    > S1;S2;

    S1=330 et S2=1100

    S1<=S2 est bien vérifié dans l'exemple...désolé de poluer

    Cdt
  • Si tous les termes sont de même signe, c'est évident.
    Il me semble que l'inégalité est moins flagrante quand la somme algébrique de tous les termes est nulle.
  • restart: S1:=0;S2:=0;
    > E:={-16,-7,-8,15,3,13};
    > for x in E do
    > for y in E do
    > S1:=abs(x-y)+S1; S2:=abs(x+y)+S2 ;
    > od;od;
    > S1;S2;


    S1=456 , S2=476

    c'est vrai que lécart S1-S2 est assez petit dans ces cas mais pas nul...

    Je fais des essais pour avoir peut etre une piste de recherche .
  • Siméon écrivait:
    > Remarque : il est facile de voir que $\sum_{i,j}
    \|x_i - x_j\| \leq 2\sum_{i,j} \|x_i + x_j\|$ est
    > vraie en toute dimension.

    Y -a -il vraiment un rapport avec la question ?? c'est très différent avec le coefficient 2...

    cordialement
  • On a évidemment l' égalité si tout x_i a son opposé parmi les x_j.
  • jacquot a écrit:
    D'où la sors-tu ?
    Quelqu'un m'a demandé si je savais la démontrer. J'ignore malheureusement d'où lui la tient...
    Je me permets tout de même une remarque : elle entraîne très simplement l'inégalité
    $$
    \sum_{i,j} |x_i + x_j| \geq n \sum_i |x_i|
    $$
    qui a fait l'objet de plusieurs problèmes d'olympiade.

    jacquot a écrit:
    Juste cette précision : l'indiciation est elle correcte ici ? $\sum_{(i,j)\in \{ 1,\dots n\} ^2 } |x_i - x_j| \leq \sum_{(i,j)\in \{ 1,\dots n\} ^2 } |x_i + x_j|$
    C'est tout à fait ça. Désolé pour la confusion.

    @acetonik : j'indiquais seulement l'inégalité avec la constante $2$ parce qu'elle plus simple à établir et vraie de façon générale.
    Comme tu le pressens, on peut montrer qu'il y a égalité dans le cas réel si et seulement si la distribution des $x_i$ est symétrique par rapport à $0$.
  • Cette mystérieuse inégalité est expérimentalement vraisemblable en dimension quelconque et pour bon nombre de normes :
    n=10
    d=20
    M=1000
    p='inf'
    Q=zeros(1:M)
    for k=1:M
        D=0
        S=0
        a=grand(d,n,'unf',-1,1)
        for l=1:n
            for m=1:l-1
                D=D+norm(a(:,l)-a(:,m),p)
                S=S+norm(a(:,l)+a(:,m),p)
            end
        end
        for l=1:n
            S=S+norm(a(:,l),p)
        end
        Q(k)=D/S
    end
    
    clf
    plot(1:M,Q)
    
  • Bonsoir,

    Une idée: quitte à oublier les termes se répétant deux fois, on veut montrer : \[
    \sum_{i<j}|x_i-x_j| \leq \sum_{i<j}|x_i+x_j| + \sum_i |x_i|
    \] L'inégalité étant invariante par permutation de deux termes, elle l'est par l'action de $\mathop{\mathfrak{S}}_n$ et on peut supposer $x_1>x_2>x_3>...>x_n$ (si tel n'est pas le cas, on permute pour obtenir une suite croissante, ça ne change rien à l'inégalité).
    Des termes se compensent dans le membre de gauche et j'intuite des cas $n=2,3,4,5$ : \[
    \sum_{i<j}|x_i-x_j|= \sum_{i=1}^{E(\frac{n}{2})} (n+1-2i)(x_i-x_{n-i+1})
    \]
  • Bonjour,
    Comme Balthazard, je réfléchis à une démonstration de l'inégalité avec les valeurs absolues.
    Cette démonstration n'est pastout à fait aboutie. En voci l'idée:

    Soient $x$ et $y$ deux réels. je note $a$ et $b$ leurs valeurs absolues, éventuellement dans le désordre, de sorte que $a>b$.
    Si $x$ et $y$ sont de même signe alors $|x+y|=a+b$ et $|x-y|=a-b$ donc $|x+y|\geqslant |x-y|$ et leur différence est $2b$.
    Si $x$ et $y$ sont de signes opposés alors $|x+y|=a-b$ et $|x-y|=a+b$. Dans ce cas,$|x+y|\leqslant |x-y|$ et leur différence est aussi $2b$.

    Je considère à présent une famille $x_1,x_2, \dots ,x_n$ de réels non nuls (le fait d'ajouter des termes nuls ne change en rien la validité ou la non-validité de l'inéquation de Siméon).
    Quitte à les réindicier, j'admets qie $|x_1|\leqslant |x_2|\leqslant \dots\leqslant |x_n|$

    Je note $N$ le nombre d'éléments négatifs et $P$ le nombre d'éléments positifs de cette famille, de sorte que $N+P=n$.
    Je peux supposer que $P\geqslant N$, quitte à changer tous les signes dans la famille, ce qui ne modifie en rien la validité ou non-validité de l'inégalité de Siméon...
    Alors, dans les sommes $\sum_{i,j}|x_i-x_j|$ et $\sum_{i,j}|x_i-x_j|$ contiennent chacune $n^2$termes dont $N^2+P^2$ termes avec $x_i$ et $x_j$ de même signe et $2NP$ termes avec $x_i$ et $x_j$ de signes opposés.
    Les premiers tendent à valider l'inégalité de Siméon, tandis que les termes de signe contraire tendent à l'invalider.
    Il est clair que les premiers sont plus nombreux que les seconds, puisque pour tous $N, P$ : $(N-P)^2\geqslant 0\Rightarrow N^2+P^2\geqslant 2NP$

    Ils sont plus nombreux certes, mais sont-ils assez forts pour imposer leur loi ?
    C'est là que ma démostration achoppe encore...

    Parmi les $2NP$ cas défavorables, il y a au moins $2N$termes en $x_1$ et voilà je me perds un peu dans la suite de mon raisonnement..
    je voudrais montrer que pour chacun des $2NP$ cas défavorables, on peut trouver un cas favorable parmi les $N^2+P^2$ qui le neutralise...
    Si on classe les $2NP$ cas défavorables suivant les $\inf (x_ix,_j)$ croissants, il me semble qu'on pourra juxtaposer à chacun de ces $2NP$ cas l'un des $N^2+P^2$ cas qui dégage au moins autant de bénéfice pour neutraliser la perte...

    Mais ça reste à préciser...


    Ta démonstration, Siméon, ressemble-t-elle à cela ?
    Quoiqu'il en soit , ce raisonnement ne me semble pas adaptable aux dimensions supérieures.
    Amicalement. jacquot
  • Idée que je n'ai pas rédigée, donc je ne suis pas sûr : on se ramène par une récurrence au cas où les $x_i$ sont tous distincts. La différence des deux membres est une fonction affine en $x_n$, donc on se ramène à montrer l'inégalité lorsque $x_n$ tend vers $\pm\infty$ ou lorsque $x_n$ est sur l'un des noeuds. Donc on peut supposer que $x_n=-x_k$ pour un certain $k$. En raisonnant de même sur les autres $x_i$, on peut supposer que les $x_i$ se regroupent deux par deux en un réel et son opposé, et ce cas particulier est un cas d'égalité.

    Mais cela ne permet pas de démontrer l'inégalité en dimension supérieure.
  • @jacquot : il reste un petit truc technique, mais tu as essentiellement l'idée de la preuve.

    Étape 1. on remarque que $|x_i+x_j|-|x_i-x_j| = 2\min(|x_i|,|x_j|)\times \begin{cases}1 & \text{if } x_i x_j > 0\\ -1 & \text{if } x_i x_j < 0\end{cases}$.

    Étape 2. Soit $A_t = \{i : x_i > t\}$ et $B_t = \{i : x_i < -t\}$. D'après l'étape 1, on a
    $$
    \frac{1}{2} \sum_{i,j} (|x_i + x_j| -|x_i-x_j|) = \sum_{i,j \in A_0} \min(|x_i|,|x_j|) +\sum_{i,j \in B_0} \min(|x_i|,|x_j|) -2 \sum_{i\in A_0,j\in B_0} \min(|x_i|,|x_j|)
    $$
    Étape 3. On utilise "l'astuce" $\min(|x_i|,|x_j|) = \int_0^\infty 1_{t < |x_i|}1_{t < |x_j|}dt$ et le fait que$\sum_{i} 1_{t < x_i} = \sum_{i \in A_t} 1 = |A_t|$ (idem pour $B_t$).
    \begin{gather*}
    \sum_{i,j \in A_0} \min(|x_i|,|x_j|) = \int_0^\infty |A_t|\cdot |A_t|\,dt\\
    \sum_{i,j \in B_0} \min(|x_i|,|x_j|) = \int_0^\infty |B_t|\cdot |B_t|\,dt\\
    \sum_{i\in A_0, j\in B_0} \min(|x_i|,|x_j|) = \int_0^\infty |A_t|\cdot |B_t|\,dt\\
    \end{gather*}
    Étape 4. En mettant tout ensemble, on a prouvé que
    $$
    \frac{1}{2} \sum_{i,j} (|x_i + x_j| -|x_i-x_j|) = \int_0^\infty (|A_t|-|B_t|)^2\,dt \geq 0.
    $$
  • Remarquons au passage que la preuve s'adapte immédiatement au cas général suivant : si $X$ et $Y$ sont des variables aléatoires réelles indépendantes et de même loi avec $\Bbb E(|X|)< \infty$, on a $\Bbb E(|X-Y|) \leq \Bbb E(|X+Y|).$
  • Tu es gentil, Siméon.
    Ta maîtrise technique te permet une présentation claire qui vaut bien mieux que mes explications confuses.
    Je me réjouis du renfort de JLT pour l'étude de ces questions intéressantes.

    Je vous souhaite d'arriver à quelque chose de concluant pour les dimensions supérieures.
    Amicalement. jacquot
  • En dimension $n\geqslant 2$, il semble assez intuitif que les situations le moins favorables sont celles où les points sont équirépartis autour de $0$
    En effet, si on fait une translation de la distribution, le membre de gauche reste invaraiant tandis que le membre de droite aura un minimum.
    Faut-il introduire l'isobarycentre ou un autre point caractéristique (point de Fermat pour un triangle...) ?
  • Bonjour Siméon and co,

    Ton inégalité n'est pas systématiquement vraie dans $\C$:
    prenons $x_1=a=4+3i$ ; $x_2=b=-4+3i$ ; $x_3=c=-6i$
    Alors $a+b=6i$ ; $b+c=-4-3i$ ; $a+c=4-3i$
    et $a-b=8$ ; $b-c=-4+9i$ et $a-c=4+9i$
    file.php?4,file=29855
    On calcule : $\sum_{i,j} |x_i - x_j|= 2\times 8 + 4\sqrt {97}\approx 55,4$
    tandis que $ \sum_{i,j} |x_i + x_j| = 3\times 6 +6\times 5 =48$.29855
  • Tu es sûr de ton calcul ?
    -->x=[4+3*%i, -4+3*%i, -6*%i]
     x  =
     
        4. + 3.i  - 4. + 3.i  - 6.i  
     
    -->s=0;for i=1:3;for j=1:3;s=s+norm(x(i)-x(j));end;end;s
     s  =
     
        55.395431  
     
    -->s=0;for i=1:3;for j=1:3;s=s+norm(x(i)+x(j));end;end;s
     s  =
     
        64.  
     
    
  • ::o J'ai oublié de doubler pour les indices égaux $x_i+x_i= 2 x_i$
    Il faut donc modifier comme suit:
    $ \sum_{i,j} |x_i + x_j| = 4\times 6 +4\times 5 =64$.

    Et l'inégalité n'est pas mise en défaut
  • Bonsoir à vous deux,

    Je suis heureux de voir que ce fil n'est pas mort. Pour ma part je n'ai pas eu le temps de réfléchir à la question ces derniers jours mais je suis toujours intéressé par quelque progrès/conjecture sur le sujet.
  • Hélas, Siméon,

    Pas de nouveauté remarquable, mais je réfléchis à ton inégalité dans $\C$ (ou dans $\R^2$ muni de la norme euclidienne.

    On peut déjà remarquer que si la famille $(x_i)$ contient des éléments nuls ou opposés, on peut les oublier, puisque leur contribution à l'augmentation du membre de gauche de ton inégalité égale leur contribution à l'augmentation du membre de droite.

    Dans $\C$, l'inégalité est évidemment vraie pour les familles de un ou deux nombres.
    Je m'intéresse à présent aux familles de trois nombres.
    Soient $a$; $b$; $c$ trois complexes et $A$; $B$; $C$ leurs représentations dans le plan
    Le membre de gauche de ton inégalité égale le double du périmètre du triangle $ABC$. Il est invariant par translations
    Le membre de droite égale $2(|a|+|b|+|c|+|a+b|+|b+c|+|c+a|)$
    Pour un triangle $ABC$ donné, j'aimerais savoir pour quel point $O$ cette somme est minimale.
    $|a|+|b|+|c|$ est minimal pour le point de Fermat du triangle
    $|a+b|+|b+c|+|c+a|$ est minimal pour le point de Fermat du tiangle des milieux du triangle ABC.
    Ces deux points sont dans une homothétie de rapport -1/2 par rapport au centre de gravité $G$ du triangle, mais GeoGebra me montre que G n'est pas forcément la position qui minimise notre somme..

    Faut-il poser la question à nos éminents géomètres?

    PS. J'ai posé la question sur le forum de Géométrie
  • Bonjour,

    Après un peu plus de réflexion, une autre piste de démonstration, celle de Siméon étant trop astucieuse à mon goût. Par homogénéité de l'inégalité, cela revient à montrer que
    \[
    F(x)=\sum_{i,j} |x_i-x_j|
    \]
    admet comme maximum $1$ sur l'ensemble $E= \left\{ \sum_{i,j}|x_i+x_j| \leq 1 \right\}$. Si on consid\`ere $x$ comme un \'el\'ement de $\mathbb{R}^n$, on peut voir $E$ comme la boule unit\é associée \`a la norme donnée par l'expression $N(x)= \sum_{i,j}|x_i+x_j|$. En particulier, $E$ est convexe, et admet même un nombre fini de points extrémaux, et comme F est convexe, c'est en l'un de ces points que son maximum est atteint. L'avantage de cette démarche c'est que le début de l'argument se généralise au cas d'une norme quelconque en dimension quelconque; le désavantage c'est que si on se place en dimension 2 et qu'on renplace la valeur absolue par la norme 2 par exemple, je n'ai pas l'impression qu'il y ait encore un nombre fini de points extrémaux.
  • Bonjour,

    Si $E_k$ est un (multi)ensemble fini de points de $\R^k (k \in\N^*)$, l'application $D_{E_k}$ qui à tout point $M$ de $\R^k$ associe la somme des distances de $M$ à chacun des points de $E_k$ admet un minimum. Appelons "Fermat $(E_k)$" l'ensemble des points de $\R^k$ en lesquels
    $D_{E_k}$ atteint ce minimum.
    Je me demande quels sont les $(k, E_k)$ tels que Fermat $(E_k)$ soit un singleton.
    En espérant que ma question n'est pas sotte...
    Amicalement,
    Paul
  • Euh...
    je retire ma question!
    Paul

    Edit:
    Soit ABC un triangle, A'B'C' son triangle médian, F et F' leurs points de Fermat.
    Par homothétie on a 2(F'A'+F'B'+F'C') = FA+FB+FC et donc FA+FB+FC+2(F'A'+F'B'+F'C')=2( FA+FB+FC ).
    Or, par l'inégalité triangulaire appliquée trois fois AB+BC+CA$\leq$2( FA+FB+FC ).
    Mais, par définition du point de Fermat, on a, pour tout point O,
    et FA+FB+FC $\leq$OA+OB+OC et F'A'+F'B'+F'C' $\leq$OA'+OB'+OC'.
    Finalement, AB+BC+CA $\leq$2( FA+FB+FC )=FA+FB+FC+2(F'A'+F'B'+F'C')$\leq$OA+OB+OC+2(OA'+OB'+OC').

    Cordialement
    Paul
  • Merci Paul,
    J'étais assez près de la solution, mais obnubilé par la recherche du point O qui minimise cette somme.
    En fait, il suffisait de montrer que, pour tout point O du plan, on a
    OA+OB+OC+2OA'+2OB'+2OC' > AB+BC+CA

    À nos lecteurs:
    Si on multiplie les deux membres de cette inégalité par 2, on a établi l'inégalité de Siméon dans le cas particulier dune famille de trois points du plan.
    (Faire une figure si nécessaire, voir aussi Forum de Géométrie)
    D'ailleurs ça reste vrai pour les dimensions supérieures, puisqu'un triangle définit un plan.
    C'est un petit progrès encourageant (tu).

    En toute rigueur, il conviendrait de traiter le cas du triangle obtusangle ayant un angle de plus de 120°, mais je crois que le raisonnement de Paul y reste applicable puisque le point de Fermat est alors sur ce sommet...

    @ suivre. Notons que pour un quadrilatère convexe, la somme des distances au sommet est minimale à l'intersection des diagonales...
    Amicalement. jacquot.
  • C'est très joli @jacquot & @pdepasse ! Malheureusement je n'ai pas l'impression que l'argument se généralise avec plus de points.

    @Balthazard, je n'ai pas très bien compris où tu voulais en venir : as-tu une preuve trouvé une autre preuve en dimension 1 ?
  • Bonjour,

    Pour quatre points A,B,C,D,

    on dirait qu'il suffit d'ajouter les quatre inégalités démontrées pour le cas de trois points (dans $ABC,BCD,CDA,DAB$)
    $AB+BC+CA\leq MA+MB+MC+2(MH+MK+MG)$ (où $H,K,G$ sont les milieux de$ BC,CA,AB$)...
    Après, méfiance quand ça paraît trop simple!

    Amicalement
    Paul

    PS:si c'est juste, la récurrence sur le nombre de points risque de marcher tranquillement
  • J'y avais pensé, mais je crois que ça ne marche pas aussi bien qu'on pourrait l'espérer : quand on prend les quatre triangles chaque arête (donc chaque milieu) est répétée deux fois, mais les sommets eux sont répétés trois fois. Avec cette méthode j'arrive seulement (en étant économe) à montrer $S \leq \frac{5}{4} S'$ au lieu de $S \leq S'$. Tu as mieux ?

    Edit : d'après ce que j'ai griffonné au brouillon, ta technique conduit en général à une inégalité du type $S \leq \frac{n+1}{4} S'$ pour $n \geq 3$, ce qui reste meilleur que $S \leq 2S'$ tant que $n \leq 7$.
  • Hélas Paul,
    Comme l'écrit Siméon, le compte n'y est pas:

    Soient $G,HJ,I,J,K,L$ les milieux de $AB,BC,CD,AD,AC,BD$
    Tes inégalités s'écrivent:
    $AB+BC+AC\leqslant MA+MB+MC+2(MG+MH+MK)$
    $AB+BD+AD\leqslant MA+MB+MD+2(MG+ML+MJ)$
    $AC+CD+AD\leqslant MA+MC+MD+2(MK+MI+MJ)$
    $BC+CD+BD\leqslant MB+MC+MD+2(MH+MI+MJ)$
    Par addition , il vient:
    $2(AB+AC+AD+BC+BD+CD)$
    $\leqslant 3(MA+MB+MC+MD)+4(MG+MH+MI+MJ++MK+ML)$

    Il y a un peu trop de $MA+MB+MC+MD$
    Caramba, on aurait aimé que ça marche.

    @ Siméon: J'ai lancé l'étude du quadrilatère sur le forum de Géométrie avec bon espoir d'aboutir.
    Cependant, nous ne pourrons pas passer notre vie à étudier des cas particuliers de plus en plus compliqués, sachant que le nombre des bipoints croît en raison de $C_n^2$:S

    Amicalement. jacquot
  • J'ai trouvé ! enfin, je crois.
    On peut déduire directement le cas euclidien de dimension $d > 1$ (éventuellement infinie quitte à se placer dans l'espace vectoriel engendré par les $x_i$) du cas $d = 1$.
    En fait c'est assez simple. Il suffit de remarquer que pour une certaine constante $C_d > 0$, on a
    $$
    \forall u \in \Bbb R^d, \qquad \| u \| = C_d \int_{\mathbb{S}^{d-1}}\, |\langle u, v \rangle| \, m(dv)
    $$
    où $m$ est la mesure uniforme sur la sphère $\mathbb{S}^{d-1}$
    (remarque, on peut aussi prendre n'importe quelle mesure invariante par isométries vectorielles de $\mathbb{R}^d$, une gaussienne par exemple).

    On utilise alors la linéarité de l'intégrale, et hop !
  • Eh bien Siméon,
    Je suis content que tu y sois parvenu
    À première lecture, je ne comprends pas, notamment ce que tu fais de ta constante Cd . Est-ce le rayon de la boule ?

    Mais je partage quand même un peu de ton bonheur, :)-D
    As-tu jeté un coup d'oeil à mes bagarres avec les vecteurs?

    Et nous ne savons toujours pas si cette inégalité est référencée.
    Amicalement. jacquot
  • La constante multiplicative n'est pas gênante du tout car elle ne dépend que de la dimension (on peut même la faire disparaître en changeant la mesure). Par exemple, en dimension 2 on a $C_2 = \frac{\pi}{2}$ car
    $$
    \int_{S^1} |\langle u, v \rangle|\,m(dv) = \frac{1}{2\pi}\int_{-\pi}^{\pi} \|u\| \cdot |\cos \theta|\,d\theta = \frac{2}{\pi} \|u\|.
    $$

    La preuve se fait alors comme suit ; on utilise l'inégalité en dimension 1 pour les "projetés" $\langle x_i, v\rangle$ ; l'intégrale d'une fonction positive étant positive on déduit
    $$
    \frac{1}{C_d} \sum_{i,j} (\|x_i + x_j\| - \|x_i - x_j\|) = \int_{\mathbb{S}^{d-1}} \sum_{i,j} (|\langle x_i, v\rangle + \langle x_j, v\rangle| - |\langle x_i, v\rangle - \langle x_j, v\rangle| )\; m(dv) \geq 0.
    $$

    Ce serait tout de même nettement plus satisfaisant d'avoir une vrai preuve géométrique. J'ai lu ce que tu as commencé à faire pour les quadrilatères, mais je ne crois malheureusement pas qu'on puisse relier simplement la somme des distances aux sommets pour un quadrilatère et pour son parallélogramme de Varignon.
  • Bonjour,

    j'aurais aimé être assez cultivé pour dire, avec aplomb: "Siméon, bien sûr, tu as raison!". Je n'en doute pour autant pas du tout, sentant plus ou moins confusément ce que veut dire ce que Siméon écrit!
    Quant à mon idée d'ajouter quatre inégalités, là, je ne doute absolument plus qu'elle était très mauvaise :)o.
    Je ne crois pas (mais c'est de l'ordre de la croyance) qu'il faille "couper en deux" le problème du quadrilatère, les sommets d'un côté, les milieux de l'autre: il doit bien y avoir des cas où le défaut des uns compense l'excès des autres.
    Je suis sur l'idée pour l'heure très gratuite, voire fantaisiste, qu'on peut penser à regarder le quadrilatère $Q(A,B,C,D)$ dont les sommets sont les points de Fermat des quatre triangles $ABC,BCD,CDA,DAB$: je peux toujours rêver que si la propriété est vérifiée pour $Q(A,B,C,D)$ , elle l'est pour $ABCD$ et réciproquement. Or quand on itère la fonction $Q$ on risque d'arriver vers un point ou un segment, et alors ça marcherait.
    Du délire vous dis-je!
    Amicalement
    Paul
  • Si $X$ et $Y$ sont indépendantes et de même loi à valeurs dans l'espace euclidien $E$ on a donc (bravo Siméon, j'avais cherché une représentation intégrale de la norme bien trop compliquée)
    $$\mathbb{E}\left(\|X+Y\|-\|X-Y\|\right)\geq 0.$$ Quelqu'un connait -il une application probabiliste de cette belle inégalité?
  • Très joli ! Du coup je me demande si on sait caractériser les normes qui admettent une représentation intégrale de la forme
    $$||x||=\int_{E^*} |\varphi(x)|\,\mu(d\varphi)$$
    où $\mu$ est une mesure.

    On a vu que la norme Euclidienne admet une telle représentation. Il est clair que la norme $||\cdot||_1$ aussi. Je ne vois pas pour la norme infinie.
  • En dimension 2, toutes les normes, avec $\varphi(x)=|x_1\cos \theta+x_2\sin \theta|.$ Il me semble que c'est comme ça qu'on montre que $t\mapsto e^{-\|t\|}$ est la tf transformée de Fourier d'une proba dans le plan, pour une norme arbitraire, en mélangeant des lois de Cauchy (Th de Ferguson 1967 si je me souviens bien- je manque de doc). Problème ouvert en dimensions supérieures.
  • C'est plutôt Ferguson 1962, Annals of Math. Statist. Vol 33 page 1260. Voir aussi G. Letac, Integration, mesure, analyse de Fourier et probabilités, exercices, Masson 1983.
  • Bonsoir,

    C'est vrai que la méthode est jolie. Si on essaye d'intégrer sur la boule duale, je ne vois pas. Par contre, si on intègre sur la boule unité de la norme elle-même, en prenant pour mesure $\mu$ une somme de Dirac placé en chacun des sommets pour les boules unités des normes 1 et $\infty$, on obtient en dimension $d$ : \begin{align*}
    \int_{|v| _{1}=1}|\langle u,v \rangle| d\mu &=2|u| _1 \\
    \int_{|v| _{\infty}=1}|\langle u,v \rangle| d\mu &= \sum_{\varepsilon} | \epsilon_1 u_1 + \epsilon_2 u_2+ \ldots+ \epsilon_d u_d |
    \end{align*}
    où $\varepsilon$ est un $d$-uplet parcourant $\left\{+-1\right\}^d$. Si $d=2$, comme $|x+y|+|x-y|=2 \max\left\{ |x|,|y| \right\}$, on obtient $$
    \int_{|v|_{\infty}=1}|\langle u,v \rangle| d\mu= 4|u|_{\infty}
    $$ J'espérais que la somme de droite serait encore proportionnelle à $|u|_{\infty}$ en dimension quelconque, mais ça ne semble pas être le cas.
  • Comme beaucoup, je trouve cette inégalité surprenante. J'ai donc un peu joué avec le code scilab donné par curiosité au début du fil et je pense avoir trouvé un contre exemple en dimension 3 pour la norme infini justement :
    In[1]:= x1 = {-1.8, 0.8, -1.9}
    Out[1]= {-1.8, 0.8, -1.9}
    
    In[2]:= x2 = {-2.2, -2.5, 0.06}
    Out[2]= {-2.2, -2.5, 0.06}
    
    In[3]:= x3 = {-0.7, 1.2, 2.1}
    Out[3]= {-0.7, 1.2, 2.1}
    
    In[4]:= x4 = {3.6, 1.5, 0.1}
    Out[4]= {3.6, 1.5, 0.1}
    
    In[5]:= Norm[x1 - x2, Infinity] + Norm[x1 - x3, Infinity] + 
     Norm[x1 - x4, Infinity] + Norm[x2 - x3, Infinity] + 
     Norm[x2 - x4, Infinity] + Norm[x3 - x4, Infinity]
    Out[5]= 26.5
    
    In[6]:= Norm[x1 + x2, Infinity] + Norm[x1 + x3, Infinity] + 
     Norm[x1 + x4, Infinity] + Norm[x2 + x3, Infinity] + 
     Norm[x2 + x4, Infinity] + Norm[x3 + x4, Infinity] + 
     Norm[x1, Infinity] + Norm[x2, Infinity] + Norm[x3, Infinity] + 
     Norm[x4, Infinity]
    Out[6]= 26.1
    
  • Conjecture: $t\mapsto e^{-\|t\|}$ est la transformée de Fourier d'une probabilité dans $\mathbb{R}^d$ si et seulement si $\sum_{i,j}\|x_i+x_j\|-\|x_i-x_j\|\geq 0$ pour tous $x_1,\ldots,x_n.$ A l'appui: 1) les cas $d=1, 2;$ 2) Le contre exemple d'Amtagpa et le fait que $e^{-\|t\|_{\infty}}$ pour $d=3$ n'est pas une tf (montré dans les exos de G. Letac cités plus haut).

    [Corrigé selon ton indication --JLT.]
  • Après réflexion, et compte tenu de ce qu'on sait sur les lois stables, la conjecture se réduit à : si une norme $\|t\|$ de $\mathbb{R}^d$ a la propriété de Siméon,
    càd $\sum_{i,j}\|x_i+x_j\|-\|x_i-x_j\|\geq 0$ pour tous $x_1,\ldots,x_n$,
    alors il existe une mesure positive $\mu$ sur la sphère unité $S$ telle que
    $$\|t\|=\int_S|\langle t,s\rangle|\mu(ds).$$ Hum, Schoenberg n'a pas fait ça ?
  • Bon,

    C'est fort intéressant. Effectivement ma formule ne donne pas une représentation intégrale pour la norme infinie en dimension quelconque, mais ça n'en est pas loin; numériquement je trouve avec l'expression donnée par la somme qu'en dimension $3$:

    $$
    |u|_{\infty} \leq \frac{1}{2^3} \int_{|v| _{\infty}=1}|\langle u,v \rangle| d\mu \leq 1.39 |u|_{\infty}
    $$
  • @P : pas facile de trouver un contre-exemple à ta conjecture...

    @Balthazard : si j'ai bien compris ta somme avec les sommets, il semble que pour $u=(1,1,1)$ on a un facteur $\frac32$ à droite, et je crois que c'est le maximum. Tu confirmes ?
  • Note : si quelqu'un remarque une erreur grossière dans ce qui suit, je lui serai éternellement reconnaissant de me l'indiquer.

    Chers amis,

    Merci à tous de vous intéresser à ce problème ! Grâce à un ami j'ai eu quelques idées nouvelles sur le sujet. En fait, j'ai (je crois) obtenu quelque chose de nettement plus général.

    Commençons par la version géométrique : soient $A$ et $B$ deux ensembles finis de points dans $\Bbb R^k$. Alors
    $$
    \sum_{a \in A, b \in B} \frac{d(a,b)}{|A|\cdot |B|} \geq \frac{1}{2} \sum_{a\in A, a'\in A} \frac{d(a,a')}{|A|^2} + \frac{1}{2} \sum_{b\in B,b'\in B} \frac{d(b,b')}{|B|^2}.
    $$
    (à illustrer en coloriant de couleurs différentes les segments correspondant respectivement au membre de gauche et au membre de droite). D'après ce qui précède, on peut prendre pour $d$ une distance euclidienne ou toute distance découlant d'une norme qui admet un écriture intégrale comme il a été question plus haut. À vrai dire, j'avais l'impression que ceci pouvait se ramener à un problème purement combinatoire et rester valable dans n'importe quel espace métrique mais le contre-exemple pour la norme $\|\cdot\|_\infty$ par @Amtagpa semble indiquer le contraire. On peut vérifier à la main que le résultat reste vrai pour la distance discrète.

    Les coefficients de poids $1/|A|^2, \dots$ peuvent être remplacés par une distribution de probabilité. Par exemple : si $X,Y$ sont des vecteurs aléatoires indépendants ayant un moment d'ordre $1$, alors
    $$
    \Bbb E(\|X-Y\|) \geq \frac{1}{2} \Bbb E(\|X-X'\|) + \frac{1}{2}\Bbb E(\|Y-Y'\|)
    $$
    où $X'$ et $Y'$ sont respectivement des copies indépendantes de $X$ et $Y$. L'inégalité de "symétrie" dont il a été question dans la discussion se retrouve en prenant $Y = -X'$.

    J'ignore toujours si ces résultats sont connus, donc si quelqu'un a une info à ce sujet...
  • @Amtagpa
    C'est ça! J'avais envoyé des vecteurs aux hasards pour avoir une idée de l'optimum mais c'est le vecteur $(1,1, \ldots,1)$ qui a l'air le meilleur en toute dimension.
    D'ailleurs on obtient alors pour la somme, en regroupant les termes ou apparaissent un seul $-1$, puis deux, puis trois... \[
    \frac{1}{2^d}\sum_{\varepsilon \in \left\{\pm 1\right\}^d} | \epsilon_1 + \epsilon_2 + \cdots+ \epsilon_d | = \frac{1}{2^{d-1}}\sum_{k=0}^{E(\frac{d}{2})}(d-2k) \binom{d}{k}
    \] Mes expérimentations numériques et l'OEIS semblent indiquer que pour $p\geq 1$
    si $d=2p$ ou $2p-1$ cette somme vaut $ \dfrac{(2p-1)!}{4^{p-1}(p-1)!^2}$.

    @Siméon
    De mieux en mieux. Si cette inégalité est encore inconnue, elle mérite clairement le nom d'"inégalité de Siméon le poisson".
  • Pardonnez-moi,

    Vous roulez en Formule 1 et j'essaye de vous suivre avec mon vélo...
    J'ai relu la démonstration de Siméon, et voici comment je la comprends (en dimension 2) :

    file.php?4,file=29955

    Il s'agit de montrer que la somme des longueurs des segments rouges est supérieure à la somme des longueurs des segments bleus.
    Siméon projette la figure sur une droite verte passant par O.
    La longueur du projeté d'une somme de 2 vecteurs est la somme des longueurs des projetés des 2 vecteurs.
    L'inégalité souhaitée est vraie pour les projetés des segments sur la droite, puisque Siméon a établi son inégalité dans l'espace de dimension 1.

    Il fait alors tourner la droite verte autour du point O.
    Pour un segment donné, il considère la longueur moyenne des longueurs de ses projetés sur un tour complet. Cette moyenne égale 2/pi fois la longueur (euclidienne) du segment. Comme l'inégalité est vraie pour les projetés quelle que soit la direction de la droite verte, elle sera vraie "en moyenne" et vraie pour les segments.

    Peut-on approuver cette description? Corriger si nécessaire.
    Si oui , elle pourra être utile à d'autres lecteurs qui ont eu du mal à comprendre cette belle démonstration synthétique.
    Amicalement. jacquot29955
Connectez-vous ou Inscrivez-vous pour répondre.